Find the minimum.

Level 1

If a a and b b are rational positive numbers such that a + b = 1 a+b=1 ,find the smallest possible value of ( 1 + 1 a ) ( 1 + 1 b ) (1+\dfrac{1}{a})(1+\dfrac{1}{b}) .


The answer is 9.

This section requires Javascript.
You are seeing this because something didn't load right. We suggest you, (a) try refreshing the page, (b) enabling javascript if it is disabled on your browser and, finally, (c) loading the non-javascript version of this page . We're sorry about the hassle.

1 solution

Lorenc Bushi
Jan 3, 2014

Expanding the brackets and we get :

( 1 + 1 a ) ( 1 + 1 b ) = 1 + 1 a + 1 b + 1 a b = 1 + a + b a b + 1 a b = 1 + a + b + 1 a b (1+\frac{1}{a})(1+\frac{1}{b})=1+\frac{1}{a}+\frac{1}{b}+\frac{1}{ab}=1+\frac{a+b}{ab}+\frac{1}{ab}=1+\frac{a+b+1}{ab}

Since a + b = 1 a+b=1 then it follows that ( 1 + 1 a ) ( 1 + 1 b ) = 1 + 2 a b (1+\frac{1}{a})(1+\frac{1}{b})=1+\frac{2}{ab} ,Thus,we must find the minimum of this expression.In order to find the minimum of this expression we must first find the maximum of a b ab .Since we know that a + b = 1 a+b=1 the first thing that comes into my mind is A M G M AM≥GM inequality.Let's take the specific numbers a 2 a^2 and b 2 b^2 and let's apply to them the A M G M AM≥GM inequality.We should have:

a 2 + b 2 2 a 2 b 2 \frac{a^2+b^2}{2}≥\sqrt{a^2b^2} , and by speeding things up we get:

a 2 + b 2 2 a b a^2+b^2≥2ab .Rewrite the left side and we have:

( a + b ) 2 2 a b 2 a b (a+b)^2-2ab≥2ab \rightarrow 1 2 a b 2 a b 1-2ab≥2ab \rightarrow a b 1 4 \boxed{ab≤\frac{1}{4}} , and by substituting this value for a b ab we get the minimum of our expression which is 1 + 2 1 4 = 9 1+\frac{2}{\frac{1}{4}}=\boxed{9}

I managed to make it right only in my third attempt. I wonder where my other answers went wrong .

Elaborately speaking , my 1 s t 1st approach to the problem was :

Applying A . M G . M A.M \geq G.M to the sets : ( 1 a , 1 ) , ( 1 b , 1 ) \left(\frac{1}{a},1\right),\left(\frac{1}{b},1\right) and ( a 2 , b 2 ) (a^2,b^2) separately we get,

1 a + 1 2 1 a ( i ) \frac{1}{a}+1\geq 2\sqrt{\frac{1}{a}}---(i)

1 b + 1 2 1 b ( i i ) \frac{1}{b}+1\geq 2\sqrt{\frac{1}{b}}---(ii)

a 2 + b 2 2 a b ( i i i ) a^2+b^2\geq 2ab---(iii)

Now, ( i ) × ( i i ) (i) \times (ii) gives,

( 1 a + 1 ) ( 1 b + 1 ) 4 1 a b ( i v ) \left(\frac{1}{a}+1 \right)\left(\frac{1}{b}+1 \right) \geq 4\sqrt{\frac{1}{ab}}---(iv)

To get the minimum value of the Left hand expression,clearly the a b ab has to be maximised as much as possible.

From ( i i i ) (iii) and from the given info( a + b = 1 a+b=1 ),it's easy to deduce the fact that :

a b 1 4 , ab \leq \frac{1}{4}, i.e the maximum value of a b ab is 1 4 \frac{1}{4} .

Hence, the minimum value of the ( 1 a + 1 ) ( 1 b + 1 ) \left(\frac{1}{a}+1 \right)\left(\frac{1}{b}+1 \right) occurs at a b = 1 4 ab=\frac{1}{4} which is 8 \boxed{8} [Clearly achievable by putting value of a b ab in the right hand expression of ( i v ) ] . (iv)].

My 2 n d 2nd approach was expanding the Left hand expression of ( i v ) (iv) and getting the expression :

2 + a b a b 4 1 a b \frac{2+ab}{ab} \geq 4\sqrt{\frac{1}{ab}}

Since,our job is to find the minimum value of the expression, it's clearly equivalent to find when the equality exists or precisely for what value of a b ab equality exists. So,squaring both sides,cancelling a b ab from the denominators as clearly a b 0 ab \neq 0 ,we get a quadratic equation where variable is a b ab .Solving the equation yields a b = 6 ± 4 2 ab=6 \pm 4\sqrt{2} . Hence we need to take just a b = 6 + 4 2 ab=6+4\sqrt{2} as a b ab has to be more. So, putting this value of a b ab in Right hand expression of ( i v ) (iv) we get the minimum value of ( 1 a + 1 ) ( 1 b + 1 ) \left(\frac{1}{a}+1 \right)\left(\frac{1}{b}+1 \right) as 2 ( 2 2 ) \boxed{2(2-\sqrt{2})} . So,I would be glad if someone point out my mistakes if there are any.

Thanks.

Bhargav Das - 7 years, 5 months ago

Log in to reply

In your first method, in (i) equality occurs at a = 1, in (ii) equality occurs at b = 1 ....since a=1, b=1 NOT possible simultaneosly (as a+b given as 1), your equation (iii) is not strict: i mean equality in (iii) is not achievable.

Such is not problem with solution by Lorenc.

To summarize: "whether Equality is achievable or not" needs always be checked in questions of finding Minimum or Maximum value using Inequalities.

Piyushkumar Palan - 7 years, 5 months ago

This was precisely my first attempt.... :p ....Cheers

Eddie The Head - 7 years, 4 months ago

Awesome question! Were you the one who made it? BTW, Nice solution.:)

Priyansh Sangule - 7 years, 5 months ago

0 pending reports

×

Problem Loading...

Note Loading...

Set Loading...